Why not E?
Why would B be the correct answer, rather than E? Thank you so much!! (LSAT Dec 2016 Logic Ga...
samlhoover on May 15, 2020
  • December 2016 LSAT
  • SEC3
  • Q8
1
Reply
hello what does it mean by adjacent chapters???
Thank you in advance!
Masada on May 8, 2020
  • December 2016 LSAT
  • SEC3
  • Q6
3
Replies
Test 80 dec 2016
logic game 6-11 Literally clueless on how to set this up
Lizkringer on April 1, 2020
  • December 2016 LSAT
  • SEC3
  • Q6
3
Replies
explanation that helped me understand this ques...
Question #3. can someone explain the right answer?
ulino23 on January 31, 2020
  • December 2016 LSAT
  • SEC3
  • Q3
1
Reply
Help
Could I please get an explanation for this question? Couldn't find the scenario that wouldn't wor...
hannahnaylor5 on January 8, 2020
  • December 2016 LSAT
  • SEC3
  • Q20
1
Reply
Why does M have to be on the red team?
I know that L and J have to be on the grean team and that O has to be on the red team. I also kno...
MACZ on January 5, 2020
  • December 2016 LSAT
  • SEC3
  • Q4
1
Reply
facilitator
Hello, I am not sure how to work with facilitators and to answer this question. I would need to ...
ca_teran1@yahoo.com on December 3, 2019
  • December 2016 LSAT
  • SEC3
  • Q2
1
Reply
first 3 variables cut off
the first 3 variables are cut off while taking the full test. pls fix
bkasra on October 29, 2019
  • December 2016 LSAT
  • SEC3
  • Q12
1
Reply
Game 4
Pe
Samir-Ghani on October 24, 2019
  • December 2016 LSAT
  • SEC3
  • Q19
2
Replies
Speed
How do you do this game fast?
Meredith on October 23, 2019
  • December 2016 LSAT
  • SEC3
  • Q19
1
Reply
how do you arrive at the answer to this question
Please share guidance on how to arrive at the answer to these type of questions on logic games? P...
Tony on August 24, 2019
  • December 2016 LSAT
  • SEC3
  • Q11
1
Reply
Can you please help with the set up of this game?
I had no idea where to start when attempting this.
Bryan on June 4, 2019
  • December 2016 LSAT
  • SEC3
  • Q19
1
Reply
help with this game setup
Hi can you help with this set up?
kadipin25 on January 26, 2019
  • December 2016 LSAT
  • SEC3
  • Q12
3
Replies